PANCE

अब Quizwiz के साथ अपने होमवर्क और परीक्षाओं को एस करें!

hyponatremia

less than 135

hypotonic serum osmolality

less than 280

A 62-year-old male is brought to the emergency department after having sustained a syncopal episode at home. Vitals include BP of 95/64 mm Hg, pulse rate of 115 bpm, and respiratory rate of 20 bpm. He is complaining of lower back pain; examination reveals a femoral pulse discrepancy and pulsatile abdominal mass. What is the most likely diagnosis? a.Acute myocardial infarction b.Acute pulmonary embolus c.Dissecting abdominal aortic aneurysm d.Wolff-Parkinson-White syndrome e.Thoracic aortic aneurysm

A classic triad of hypotension, pulsatile abdominal mass, and flank or back pain has been described but not present in every patient with dissecting abdominal aortic aneurysm. The vast majority of abdominal aortic aneurysms are asymptomatic until expansion or rupture. The presence of unexplained syncope in a male patient over the age of 60 years should raise the possibility. Acute myocardial infarction presents with chest pain with or without dyspnea, nausea, or vomiting. The pain typically radiates to the jaw or left arm. Acute pulmonary embolus presents with sudden dyspnea and chest pain, which is sensitive to inspiration. Wolff-Parkinson-White syndrome is an arrhythmia characterized by a wide QRS and delta wave (slur) on ECG; clinical manifestations include palpitations, SVT, and syncope. A thoracic aortic aneurysm typically presents with widened mediastinum on CXR and is associated with patients who have underlying connective tissue disease such as Marfan and Ehlers-Danlos syndromes.

A 71-year-old male presents with right-sided hemiparesis for 1 hour. He had been sitting at his desk when he suddenly had difficulty controlling his right arm. He called for his wife who states he was yelling gibberish. He had difficulty walking. She called EMS who transported the patient to the emergency department. He is a former 1-pack-day smoker, quit 12 years ago. What abnormal speech pattern is most likely in this patient? a.Nonfluent speech, impaired repetition, and normal comprehension b.Fluent speech and markedly impaired comprehension c.Fluent speech with impaired repetition, frequent errors, but relatively preserved comprehension d.Deficits in all language functions, producing only nonword utterances e.Deficits in the production of oral speech with retention of comprehension and the ability to write

A left-sided infarction results in right hemiparesis and Broca aphasia, an expressive aphasia characterized by nonfluent speech with normal comprehensive. Fluent speech with markedly impaired comprehension is Wernicke aphasia, associated with lesions in the superior temporal gyrus, typically without motor deficit. Conduction aphasia results in fluent speech with paraphrasing errors and relatively preserved comprehension. Global aphasia (extensive perisylvian injury) is defined as deficits in all language functions; the patient cannot follow commands and is often mute or limited to guttural utterances. Pure word mutism produces deficits in the production of oral speech (dysarthria and facial paresis) with retention of comprehension and the ability to write.

With what factor is a poor prognosis of malignant melanoma most strongly associated? a.Depth of the lesion b.Location of the lesion c.Size of the lesion d.Speed of evolution e.Variation in color

A melanoma entirely within the epidermis carries a very good prognosis. As the thickness progresses beyond the epidermis, the prognosis diminishes. Location also affects the prognosis, just not as prominently as the depth, with lesions on the upper back, upper arm, neck, or scalp generally having a poorer prognosis. A larger lesion size is certainly concerning, but neither size, color, nor speed of evolution is a key to the patient's prognosis.

A 66-year-old healthy female presents as a new patient. She has no complaints, no significant past medical history, and does not smoke or drink alcohol. She states she received a pneumococcal vaccine 3 years ago and gets the flu shot annually. What is your recommendation regarding immunizations? a.After 10 consecutive seasons, annual flu vaccine is no longer needed b.Encourage pneumococcal booster every other year c.Obtain pneumococcal titers to assure protection d.Repeat pneumococcal vaccine at age 69 years

A second pneumococcal vaccine should be given 6 years after the first dose if the first dose is given in a patient younger than 65 years. The pneumococcal vaccine does not require annual boosters. Pneumococcal titers are not recommended after vaccines. Annual flu shots should be encouraged throughout life.

In the setting of community-acquired pneumonia, the presence of which of the following factors is an indication to perform Legionella urinary antigen testing? a.Active alcohol abuse b.Asplenia c.Cavitary opacities on chest x-ray d.COPD e.Fibronodular lesions in the upper lung lobes

Active alcohol abuse is a risk factor for Legionella pneumoniae as well as Gram-negative organisms such as Klebsiella. Patients are typically very ill. Legionella is more likely to show a diffuse pattern. Patients with asplenia are at higher risk of infection with encapsulated organisms, but Legionella is not an encapsulated organism. Cavitary opacities on chest x-ray are indicative of pneumococcal pneumonia. Patients with COPD are more likely to acquire pneumonia because of Gram-negative organisms or pneumococcus. Fibronodular lesions in the upper lung lobes are suggestive of tuberculosis.

A 38-year-old patient with HIV complains of pain on swallowing. Oral examination is normal. Endoscopy reveals multiple, shallow ulcers in the lower one-third of the esophagus. What is the recommended treatment? a.Acyclovir b.Lidocaine c.Fluconazole d.Ganciclovir e.Omeprazole

Acyclovir is the treatment of choice in herpes simplex esophagitis. Fluconazole would be appropriate in Candida esophagitis, which appears as white-to-yellow, diffuse linear plaques. Ganciclovir would be appropriate in CMV esophagitis, which appears as one to few, large, shallow-to-deep ulcers. Omeprazole is a proton pump inhibitor effective in GERD and PUD. Lidocaine swish-and-spit is effective analgesia for painful oral or pharyngeal lesions.

The risk of developing endometrial cancer in menopausal women that take hormone therapy to control symptoms can be significantly reduced by the administration of which of the following? a.Cyclic oral progesterones b.Estrogen patch c.GnRH agonist intramuscularly d.Oral testosterone e.Danazol capsules

Adding oral progesterone to estrogen therapy for the treatment of menopausal symptoms has been found to decrease the risk of developing endometrial hyperplasia, which leads to subsequent endometrial cancer. The use of estrogen alone has been found to cause endometrial hyperplasia, which leads to endometrial cancer. GnRH agonists (i.e., Lupron) put a woman into a pseudomenopausal state; these products are often used to treat endometriosis and uterine fibroids. Their use is not implicated in the development of endometrial cancer; however, they should be used for no longer than 6 months at a time because of risk of significant bone demineralization. Oral androgen, such as testosterone or danazol, is sometimes prescribed for decreased libido and for the treatment of endometriosis; it is not related to the development of endometrial cancer.

Which of the following is an indication to begin early screening for prostate cancer? a.African American race b.Cigarette smoking c.History of cryptorchidism d.Low sperm count e.Genital herpes

African American men have the highest incidence of prostate cancer worldwide. An African American male has a 40 times greater chance of developing prostate cancer than males in Africa. Cryptorchidism is associated with an increased risk of testicular cancer. Low sperm count is not associated with an increased risk of prostate cancer. Cigarette smoking is not associated with prostate cancer but with cancers of the mouth, lung, pharynx, esophagus, pancreas, kidney, and bladder. HPV (genital warts) is associated with increased risk of prostate cancer, but HSV (genital herpes) is not.

A 49-year-old patient with a history of hypertension presents for follow-up of a nonfasting total cholesterol of 246 mg per dL obtained during a community health fair. Which of the following is the most appropriate response? a.Provide advice on lifestyle management to improve lipid profile b.Obtain a complete lipid profile c.Reassurance that this is an acceptable nonfasting result d.Start an HMG Co-A reductase inhibitor

Although diet and exercise are to be encouraged in all patients to improve cardiovascular risk, tailored recommendations should reflect the individual patient's risk profile. Current evidence indicates that nonfasting cholesterol results are fairly consistent with fasting results (usually within 8 mg/dL), so a cholesterol level of 246 mg per dL even when nonfasting should raise concern. In order to accurately assess this patient's cardiovascular risk, a full lipid profile after 12-hour overnight fasting must be obtained before considering any course of treatment. Treatment is based primarily on clinical assessment of cardiovascular risk using the risk calculator such as AHA/ACC risk calculator.

A 58-year-old patient complains of "coffee-grounds" emesis and dark, tarry stools. There is a history of heavy alcohol use and NSAID exposure secondary to a painful musculoskeletal condition. The patient denies melena. What is the diagnostic test of choice? a.Abdominal computed tomography b.Abdominal plain film radiography c.Colonoscopy d.Upper endoscopy e.Upper gastrointestinal series

An upper endoscopy is the study of choice for suspected upper GI bleeding. Common causes include gastroduodenal ulcers, erosive esophagitis, erosive gastritis, Mallory-Weiss tears, vascular anomalies, portal hypertension, or malignancy. Computed tomography is not recommended in the workup of acute upper gastrointestinal bleeding, although may be helpful after the patient is stabilized to search for an underlying cause. Abdominal plain film radiography is useful to rule out the presence of free air under the diaphragm if there is a concern for perforation but will not diagnose the underlying etiology. Colonoscopy is indicated for lower GI bleeding. This clinical presentation suggests upper GI bleeding. Upper GI series is an acceptable alternative to diagnose neoplasms or ulcers when endoscopy is not available.

A patient is diagnosed with mild general anxiety disorder and given a prescription for lorazepam. Concomitant ingestion of which substance will cause a reduction in absorption of this drug and, therefore, should be avoided? a.Sulfa drugs b.Antacids c.Contraceptives d.Warfarin e.Vitamin K

Antacids, especially aluminum-based antacids, may slow the absorption of lorazepam and other benzodiazepines. Contraceptives commonly increase the concentration of benzodiazepines. Warfarin does not interfere with benzodiazepines but will alter prothrombin time. Sulfa drugs and vitamin K do not interact with benzodiazepines.

A slender elderly female has a long history of untreated elevated serum calcium. Which of the following historic findings is most likely also present because of her known abnormal serum calcium? a.Normal DEXA scan b.Dyspnea on exertion c.Hypertension d.Loss of thirst e.Nephrolithiasis

Approximately 20% of patients with hyperparathyroidism report a history of kidney stones, mostly of the calcium oxalate type. Chronic hypercalcemia as a result of untreated hyperparathyroidism is associated with diffuse demineralization and cystic lesions, which will cause an abnormal DEXA scan (score <−1.0 SD). Most patients with hyperparathyroidism are asymptomatic, although some patients may report polyuria and polydipsia owing to hypercalcemia-induced nephrogenic diabetes insipidus; it is not associated with dyspnea or hypertension.

A 14-year-old male presents with fever, myalgias, arthralgias, carditis, and polyarthritis. ASO titer is positive. What is the treatment of choice to relieve his fever and arthralgias? a.Acetaminophen b.Codeine c.Ibuprofen d.Naproxen e.Salicylates

Aspirin is the drug of choice in rheumatic fever; it has consistently shown greater anti-inflammatory properties over other NSAIDs or acetaminophen. It reduces fever and arthralgias significantly.

A 50-year-old male presents after an apparent suicide attempt from drug ingestion. History is unobtainable. The patient's BP is 130/70 mm Hg. He is tachycardic and displays rapid deep breathing. Laboratory results are as follows:Na+ 140 mEq per dLK+ 4.0 mEq per dLCl− 105 mEq per dLHCO3− 10 mEq per dLCreatinine 1.0 mg per dLGlucose 120 mg per dLSerum pH 7.12 Which of the following drugs was most likely taken? a.Acetaminophen b.Aspirin c.Lithium d.Phenobarbital

Aspirin overdose, although initially leading to respiratory alkalosis, subsequently causes a high anion gap acidosis. Acetaminophen, lithium, and phenobarbital do not generally cause an increase in the anion gap.

A 19-year-old female notes a 2-month history of white watery discharge from her breasts. It was scant at onset but is more obvious now. She denies pregnancy and states that she has never had sexual intercourse. Menses have been regular, and LMP was 1 week ago. Pregnancy test is negative. Physical examination is normal, except for bilateral milky nipple discharge. What other relevant review of system questions should be asked to support the most likely diagnosis? a.Anosmia, voice changes, and racing heart b.Diplopia, tinnitus, and fatigue c.Tinnitus, hearing loss, and disequilibrium d.Visual loss, headache, and fatigue e.Visual loss, hearing loss, and voice changes

Bitemporal visual loss, headache, and fatigue as a result of hypothyroidism can be characteristics of pituitary adenoma, which can lead to hyperprolactinemia and milky discharge in the nonlactating breast. Primary amenorrhea associated with diminished olfaction is associated with a pituitary deficiency (Kallmann syndrome). Tinnitus, hearing loss, and disequilibrium suggest an acoustic neuroma. Diplopia with tinnitus and fatigue with headache may indicate increased intracranial pressure. Visual loss, hearing loss, and voice changes are concocted mix of symptoms not indicative of any particular diagnosis.

A 56-year-old male is 3 months post-liver transplant. He complains of left eye floaters, blurriness, light flashes, and blind spots in his vision. The eye is red, and the retina has multiple yellow/white proliferative lesions along the periphery. What is the most likely diagnosis? a.Cytomegalovirus retinitis b.Herpes keratitis c.Optic neuritis d.Uveitis e.Conjunctivitis

CMV retinitis is most often associated with a CD4 count <50. Acquired immune deficiency via transplant or HIV infection puts people at risk owing to impaired CD4 number or cell function that enables uninhibited CMV replication. Multiple white/yellow retinal lesions are characteristic and usually occur in a perivascular distribution. Uveitis can occur concomitantly in immunocompromised patients with CMV but typically occurs in severe cases. The more typical anterior uveitis presents with pain, redness, and photophobia, and examination using slit lamp reveals the presence of leukocytes in the anterior chamber. Optic neuritis is an inflammatory, demyelinating disorder that presents as acute, monocular vision loss; and funduscopic examination is usually normal. Herpes keratitis presents with anterior pain, blurry vision, photophobia, watery discharge, and frequently a red eye. Ciliary flush and punctate or dendritic epithelial lesions are characteristic on examination. Conjunctivitis is irritation or infection on the surface of the eye and would not involve the retina.

A 9-year-old with recurrent acute otitis media presents with ear pain and purulent aural discharge for several weeks. Which of the following organisms is the most likely agent? a.Moraxella catarrhalis b.Haemophilus influenzae c.Pseudomonas aeruginosa d.Streptococcus pneumoniae e.Aspergillus

CSOM is often preceded by an episode of acute otitis media and is characterized by chronic drainage from the middle ear associated with TM perforation. Pseudomonas aeruginosa and Staph aureus are associated with CSOM as well as with acute otitis externa. Other organisms that are associated with CSOM are Proteus, Corynebacterium, and Klebsiella pneumoniae. Moraxella catarrhalis, Haemophilus influenzae, and Streptococcus pneumoniae are more commonly associated with acute otitis media. Aspergillus is implicated in fungal etiology of acute otitis externa.

A patient is 6 weeks post-aortic valve replacement. He presents with low-grade fever and malaise for 1 week. Examination reveals a new systolic murmur. Echocardiography shows large vegetations around the inferior side of the mitral valve. What is the most likely etiology? a.Candida albicans b.HACEK organisms c.Staphylococcus aureus d.Streptococcus viridans e.Clostridium difficile

Candida is a slow-growing yeast; the most common source is a contaminated line. It typically causes large vegetations in endocarditis. HACEK organisms tend to grow on native valves. Staphylococcus causes smaller vegetations; it is more common in injection drug users. Streptococcus viridans is the most common cause of endocarditis overall; it typically occurs as a late complication of valve replacement and presents as small vegetations and embolic events. Clostridium is associated with infections of the GI tract.

A 53-year-old female recently diagnosed with infective endocarditis complains of nonpainful erythematous lesions located on her palms and soles. What are these lesions? a.Erythema marginatum b.Janeway lesions c.Osler nodes d.Roth spots e.Ecthyma

Clinical manifestations of infective endocarditis may include peripheral lesions such as subungual "splinter" hemorrhages, Roth spots (retinal lesions), Osler nodes (painful violaceous lesions of the fingers, toes, or feet), or Janeway lesions (painless erythematous lesions of the palms and soles). Erythema marginatum is also associated with rheumatic fever; it presents as transient erythematous nonpainful, nonpruritic rings. Osler nodes are painful violaceous lesions of the fingers, toes, or feet seen in endocarditis. Roth spots are retinal hemorrhages seen in endocarditis. Ecthyma is a deeper form of impetigo caused by staph or strep with ulcerations occurring on the extremities.

A 50-year-old male with a past medical history of hypertension and dyslipidemia and a family history of coronary artery disease is starting atorvastatin. Which of the following medications taken concurrently could increase his risk of developing statin-associated rhabdomyolysis? a.Diltiazem b.Eprosartan c.Hydrochlorothiazide d.Lisinopril e.Propranolol

Diltiazem inhibits the cytochrome P450 liver enzymes, which metabolize most statin medications. Drugs that block this system and can, therefore, increase the risk for statin-associated rhabdomyolysis are many but include erythromycin, clarithromycin, clotrimazole, itraconazole, verapamil, and protease inhibitors. There is also an increased risk with concomitant niacin and fibric acids.

A 20-year-old male complains of increased anxiety in social settings. He realizes that he has been turning down invitations to go out with other people and prefers to be alone where he knows he will not be embarrassed by doing something "stupid." He is currently working at a factory where he works alone and can avoid interacting with other people. He avoids most social contact and has one friend from high school who he sees from time to time. What would be the most effective therapy for this patient? a.Insight-oriented therapy b.Aversion therapy c.Exposure therapy d.Cognitive-behavioral therapy e.Self-help group therapy

Cognitive-behavioral therapy is a broad therapy used to treat several psychiatric illnesses. It is particularly effective with phobias, such as the social phobia, demonstrated by the patient in this case. Exposure therapy is effective in specific phobias, such as fear of snakes. Insight-oriented therapy may be helpful in many psychiatric illnesses, including phobias, but it is not effective unless combined with behavioral therapy for patients with phobias. Self-help group therapy, such as Alcoholics Anonymous or Overeaters Anonymous, is helpful to provide support for people coping with the same problem. Supportive-group based therapy is generally not recommended for patients with social phobias. Aversion therapy is used for alcohol abuse and other disorders where impulsive behavior is involved. It is the application of a noxious stimulus after a behavioral response and is a controversial form of therapy.

A 15-year-old female with a history of well-controlled asthma presents for reevaluation. She currently uses a low-dose inhaled steroid daily. Since joining the track team 2 weeks ago, she has noticed an increased need to use her rescue albuterol inhaler. What is the recommended management for the most likely diagnosis?

Cromolyn is a mast cell modulator that inhibits both early and late responses to exercise. Administration before exercise is very effective and negates the need to increase steroid dose in a patient who is otherwise well controlled. Anticholinergic inhalers are not effective in exercise-induced asthma. A high-protein diet may help with muscle mass and endurance, but will not affect the asthma. Although activities may need to be modified based on asthma severity, all patients should be encouraged to remain active and exercise.

A 46-year-old male presents with acute flank pain and hematuria. This is the third episode in 7 months. After appropriate treatment, he should be encouraged to maintain a diet restricted in which of the following? a.Bran b.Carbohydrates c.Fats and oils d.Fluids e.Sodium and protein

E. Sodium intake increases sodium and calcium excretion and monosodium urate saturation. Protein also increases calcium, oxalate, and uric acid excretion. All of these factors can lead to stone formation. Increased fluids throughout day and night are important in reducing stone recurrence. Bran significantly lowers urinary calcium, reducing risk for stone recurrence, so it should be encouraged. Carbohydrates and fats do not have any impact on urinary stone disease.

A 59-year-old male with a history of hypertension for 20 years presents with chest pain, SOB, and palpitations worsening over the past 5 hours. On admission, he is pale and diaphoretic with a BP reading of 88/48 mm Hg and an irregular pulse rate of 138 bpm. ECG shows an irregular rhythm with no visible P waves and narrow QRS complexes. What is the immediate treatment? a.Anticoagulation b.Defibrillation c.Electrical cardioversion d.Pacemaker implantation e.Epinephrine

Electrical cardioversion is preferred in unstable patients (shock, severe hypotension, pulmonary edema, etc.) with atrial fibrillation. Although most patients who remain in controlled atrial fibrillation require anticoagulation therapy (warfarin, dabigatran, etc.) to decrease the risk of stroke, it is not an acute therapy for this patient. Defibrillation is provided to patients with no pulse. Pacemaker implantation is needed in cases of second-degree Mobitz and third-degree heart blocks, as well as for severe bradycardia and tachy-brady rhythms or sick sinus syndrome. Giving epinephrine may increase his tachycardia and is not indicated for the treatment of unstable atrial fibrillation.

What is the routine standard schedule for immunization against the hepatitis B virus? a.Baseline then 1 and 6 months later b.Baseline then 2 and 4 months later c.Baseline with booster at 2 months d.Baseline with booster at 6 to 18 months e.Baseline with booster at 12 to 15 months

Hepatitis B immunization series consists of three shots: baseline and then 1 and 6 months later. It is recommended to begin the series at birth, but, if not, the same schedule is to be followed once the series is begun. Baseline then 2 and 4 months is the schedule for DTaP, typically starting at age 2 months. Baseline with booster at 2 months is the schedule for varicella-zoster vaccine in those over 13 years of age. Baseline with booster at 6 to 18 months is the schedule for hepatitis A vaccine.

A 5-year-old male presents to the clinic with a sore throat and stomach pain. On physical examination, he has a fever of 100.6°F and a rash on his cheeks. Rash is noted to be lacy, maculopapular, evanescent on the trunk and limbs as well as a fiery red "slapped-cheek" appearance on the face. What is the likely diagnosis? a.Erythema infectiosum b.Measles c.Mumps d.Rubella e.Lupus erythematosus

Erythema infectiosum, also known as "fifth disease," presents as a fiery red "slapped-cheek" appearance, circumoral pallor, and a subsequent lacy, maculopapular, evanescent rash on the trunk and limbs. Associated symptoms are malaise, headache, pruritus (especially on the palms and soles), and diarrhea (rarely); systemic symptoms and fever mostly subside when the rash appears. A measles rash appears on the face and behind the ears with pinhead-sized papules that coalesce to form a brick red, irregular, blotchy maculopapular rash that spreads to the trunk and extremities, including the palms and soles. Mumps most commonly present with parotid tenderness and overlying facial edema. Rubella presents with a fine, pink maculopapular rash that fades from the face, trunk, and extremities in rapid progression, usually lasting 2 to 3 days. Discoid lupus erythematosus cutaneous presentation is violaceous red, well-localized, single or multiple plaques, 5 to 20 mm in diameter, usually on the face, scalp, and external ears, and is associated with atrophy, telangiectasia, central depigmentation or scarring, a hyperpigmented rim, and follicular plugging. In subacute cutaneous lupus erythematosus, the lesions are erythematous annular or psoriasiform plaques up to several centimeters in diameter and favor the upper chest and back.

A 40-year-old female presents complaining of feeling "down" for the past month. She complains that she is very tired all the time despite sleeping 9 to 10 hours per night. She states she has no energy to do anything and has stopped going out on weekends with friends owing to fatigue and loss of interest. She has also noted a 6-lb weight loss and admits to eating less in the past few weeks. She has had episodes of "sadness" in the past, but they were not as severe. The patient denies thoughts of suicide. What is the best pharmacologic treatment for this patient? a.Carbamazepine b.Fluoxetine c.Lorazepam d.Lithium carbonate e.Ergotamine

Fluoxetine is one of many 5-HT reuptake inhibitors (SSRIs) commonly used in the treatment of major depression. Lorazepam is a benzodiazepine and may be used to treat anxiety disorders and/or alcohol withdrawal. Lithium carbonate is a mood stabilizer used to treat bipolar disorder. Carbamazepine is an antiepileptic medication that may be useful in treating bipolar disorder. Ergotamine is used in the treatment of migraine; it is an older, second-line drug.

An otherwise healthy 22-year-old patient presents with mild jaundice. Which of the following additional historical factors best supports a diagnosis of Gilbert disease? a.Episodic attacks of jaundice associated with itching and malaise b.Family history of recurrent jaundice c.Family history of sickle cell trait d.History of fatty food intolerance e.Recent fever, malaise, and myalgias

Gilbert syndrome is the most common hereditary hyperbilirubinemia. With Gilbert syndrome, bilirubin levels typically do not reach levels >3 mg per dL (and, thus, the appearance of symptoms) until the patient develops an intercurrent illness or undergoes a prolonged fast. Recurrent intrahepatic cholestasis results in episodic attacks of jaundice, itching, and malaise. Patients with sickle cell disease, not trait, develop chronic jaundice from ongoing hemolysis. History of fatty food intolerance with jaundice is suggestive of acute cholangitis. The sudden appearance of jaundice in a previously healthy young person, especially if preceded by a brief prodrome of fever, malaise, and myalgias, is likely to be caused by viral hepatitis.

What is the most common manifestation of rheumatic heart disease? a.Arrhythmia b.Pericardial effusion c.Pericarditis d.Valvular defects e.Pulmonary embolus

Rheumatic heart disease most commonly affects the mitral and aortic valves.

A 35-year-old overweight female presents to the clinic complaining of tender inflammatory nodules and abscesses in her axillae. The lesions have waxed and waned over the past few years, but have become more painful and bothersome in the past month. Some of the larger lesions are draining a purulent material. What is the most likely diagnosis?

Hidradenitis suppurativa, most likely diagnosis, is a chronic disease affecting apocrine gland follicles. Those affected will develop tender, inflammatory nodules, abscesses, and sinus tracks in the axillae (more common in females) and anogenital area (more common in males). Persons with acanthosis nigricans develop symmetrical, hyperpigmented, velvety plaques in their axillae, posterior neck, and groin, due most often to hyperinsulinemia. Epidermoid cysts are slow-growing, mobile masses that occur when epidermal elements become deposited in the dermis. They can become inflamed and infected. Erythema nodosum primarily affects females, as does hidradenitis suppurativa, and patients will also develop painful nodules but primarily on the extensor surfaces of the lower legs. Various underlying etiologies have been linked to this disorder, such as infection, sarcoidosis, inflammatory bowel disease, and pregnancy. Vitiligo results in hypopigmented patches because of a loss of pigment production by melanocytes.

A 36-year-old G3P3003 female with a history of three cesarean sections presents with acute abdominal pain and distention. She is nauseous but has not vomited. Bowel sounds are hyperactive, coming in waves; abdomen is diffusely tender, no rebound. What is the most likely diagnosis? a.Acute gastroenteritis b.Ischemic bowel c.Pelvic inflammatory disease d.Perforated appendix e.Small bowel obstruction

Hyperperistalsis, which comes in waves (crescendo-decrescendo), is characteristic of early small bowel obstruction. As the condition progresses, the bowel will fatigue, resulting in lack of bowel sounds. This is a surgical emergency. Acute gastroenteritis is most commonly viral; bowel sounds will be hyperactive but steady, not coming in waves. Ischemic bowel presents with pain out of proportion to physical findings. Pelvic inflammatory disease presents with lower abdominal pain, fevers, chills, and menstrual disturbances associated with cervical motion tenderness or adnexal tenderness on examination. Perforated appendix presents with guarding and rebound.

A neonate is found to have right ventricular (RV) hypertrophy, RV outflow obstruction, and an overriding aorta via cardiac imaging. What additional finding would you expect to see in this child? a.Atrial septal defect b.Discrepancy in the upper and lower blood pressure c.Left bundle branch block d.Ventricular septal defect (VSD) e.Jaundice

In tetralogy of Fallot, anterior deviation of the pulmonary outflow results in an obstruction to the RV outflow resulting in a VSD and completes the diagnostic criteria for tetralogy of Fallot. Although electrocardiography (ECG) findings are not of great significance, right bundle branch block (not left) can occur in tetralogy of Fallot. Atrial septal defect only occurs in about 15% of patients with tetralogy of Fallot. Dissimilar blood pressure readings are pathognomonic of coarctation of the aorta. Jaundice of the newborn can be caused by several issues, including breastfeeding and prematurity, but is not seen in the tetralogy of Fallot.

A 38-year-old laboratory worker was exposed to anthrax spores. He develops fever, malaise, headache, and cough, which progresses to congestion and coryza over several hours. What is expected on the chest x-ray (CXR)? a.Bilateral hilar adenopathy b.Pleural plaque and calcifications c.Diffuse round-to-oval calcified lesions d.Mediastinal widening e.Pleural effusions and granulomatous lesions

Inhalation anthrax most typically causes mediastinal widening secondary to hemorrhage and lymphadenitis. Bilateral hilar adenopathy is most indicative of lymphoma. Diffuse round-to-oval calcified lesions are indicative of silicosis. Pleural effusions and granulomatous lesions are indicative of tuberculosis. Pleural plaques and calcifications are seen in mesothelioma.

A 68-year-old female presents with rapid onset of significant eye pain and visual loss. She notes that lights appear to have "halos." Examination reveals a red eye, steamy cornea, and dilated, nonreactive pupil. What is the initial treatment option? a.IV acetazolamide b.IV mannitol c.Topical pilocarpine d.Topical latanoprost e.Ophthalmologic prednisone

Initial treatment of primary acute angle-closure glaucoma is control of intraocular pressure and should be provided for significantly elevated pressure (i.e., >40 mm Hg). Either IV or oral acetazolamide is acceptable. Pilocarpine, a miotic used to allow excess fluid to drain from the eye, is given in combination with a regimen including agents like timolol maleate and apraclonidine. Definitive treatment, however, is with surgery (peripheral iridotomy or iridectomy). Osmotic diuretics, such as mannitol, may also be required, especially in patients with severely elevated intraocular pressure; however, first-line treatment is acetazolamide to lower the pressure and preserve vision. Ophthalmologic prednisone is used after surgery and in some cases of injury. Prostaglandin analogs like latanoprost are often used as first-line therapy in chronic open-angle glaucoma.

A 26-year-old female presents with a severe exacerbation of asthma. She is treated with oxygen, short-acting β-antagonist, ipratropium, and systemic steroids. Addition of which of the following medications will produce a detectable improvement in airflow? a.Long-acting β-agonist b.Magnesium c.Montelukast d.Theophylline e.Azithromycin

Intravenous magnesium may improve airflow in patients presenting with acute severe asthma. Long-acting β-antagonists (LABAs) are contraindicated in acute asthma. They are used in the prevention of asthma symptoms. Montelukast is leukotriene modifiable; it is not effective in acute exacerbations. Theophylline is an effective bronchodilator but has much interpatient variability and many adverse side effects. Azithromycin is an antibiotic that may be prescribed if the patient has an underlying bacterial infection, but it will not improve airflow.

An 88-year-old male with advanced dementia has become increasingly agitated at night. The family is asking for a prescription to help calm him down. An atypical antipsychotic is chosen. Which of the following side effects should be discussed with the family? a.Increased risk of stroke b.Weight loss c.Diarrhea d.Urinary incontinence e.Headache

It is common for patients with advanced dementia to have behavioral disturbances. At times, they can be display agitation and aggression. Atypical antipsychotics are sometimes prescribed for these patients. However, in patients who have vascular risk factors, there is an increased risk of stroke and an increased risk of mortality.

A 78-year-old female presents with pain and stiffness of the pelvic and shoulder girdle that has been ongoing for 3 to 4 weeks. She denies any headache, jaw claudication, or visual disturbances. Erythrocyte sedimentation rate (ESR) is 107 (0 to 30 mm/hour). What is the recommended treatment for this patient's most likely diagnosis? a.Cyclophosphamide b.High-dose prednisone c.Low-dose prednisone d.Methotrexate e.NSAIDs

Low-dose prednisone is indicated in patients with polymyalgia rheumatica. High-dose prednisone would be indicated if the patient also had signs of giant cell (temporal) arteritis—the so-called "above the neck" symptoms (jaw claudication, headache, scalp tenderness, and visual changes). NSAIDs are not effective in the treatment of polymyalgia rheumatica. Cyclophosphamide is effective in vasculitides, such as Wegener granulomatosis and microscopic polyangiitis. Methotrexate is indicated in the treatment of RA.

A 68-year-old female with 40 pack-year of tobacco use presents for evaluation of gradual painless loss of central vision bilaterally described as dark patches. She also describes distortion in her vision making straight edges appear curvy. Which of the following dilated funduscopic examination findings is most consistent with the likely diagnosis? a.Small, bright-yellow or white deposits under the retina seen in clusters b.Hyperemic, swollen optic disc with blurred margins c.Rings and crescents along the temporal border of the optic disc d.Venous tapering, flame-shaped hemorrhages, copper wiring e.Swollen optic disc

Macular degeneration is an important cause of gradual central visual loss in older adults, particularly over the age of 65 and especially smokers. Patients often complain of distortion of straight lines (metamorphopsia). AMD can be classified as dry atrophic or wet exudative with findings of drusen, which can be hard or soft in atrophic and usually larger, soft drusen or retinal pigment epithelial clumping in exudative. Drusen represent deposits of extracellular material made up of lipids and proteins. Funduscopic examination of patients with hypertensive retinopathy may reveal cotton wool patches, copper wiring, silver wiring, flame-shaped hemorrhages, AV nicking, or tapering. Rings and crescents are developmental variations of either white sclera or black retinal pigment; they are typically found along the temporal border of the optic disc. A hyperemic, swollen optic disc with blurred margins in the presence of bulging of the physiologic cup indicates papilledema. A swollen optic disc is also seen on examination of a patient with acute angle-closure glaucoma.

A 50-year-old male presents for routine health maintenance. He states that he could "stand to lose a few pounds" but is otherwise in good health. Past medical history and family history reveal no risk factors. Examination reveals abdominal obesity (waist circumference of 45 inches) and BP of 154/94 mm Hg. What additional finding would confirm a diagnosis of metabolic syndrome?

Metabolic syndrome patients are at elevated risk for cardiovascular disease and diabetes. Diagnosed by the presence of three or more of the following criteria: central obesity (waist circumference >40 inches in men and >35 inches in women), triglycerides ≥150 mg per dL, HDL cholesterol <40 mg per dL in men or <50 mg per dL in women, elevated BP (>130/85 mm Hg or taking hypertensive medications) Fasting blood glucose ≥100 mg per dL. LDL cholesterol and fasting insulin levels are not included in the criteria for metabolic syndrome.

You are conducting a funduscopic examination on a 58-year-old female patient with long-standing type 2 DM. Her most recent HbA1c returned at 7.5%. She has no other comorbid conditions. What is the most likely retinal abnormality you might find in this patient? a.Copper wire arteries b.Drusen c.Focal narrowing d.Microaneurysms e.Papilledema

Microaneurysms are generally the first findings in nonproliferative diabetic retinopathy. As the severity progresses, patients will develop dot and blot hemorrhages and cotton wool spots. Eventually, more serious changes can occur, such as hard exudates and neovascularization. Copper wire arteries are characteristic of hypertensive retinopathy and result from the light reflex giving off a bright coppery luster when an artery becomes full and tortuous. Drusen, yellowish or white spots, are most often associated with normal aging but can be seen in pathologies such as macular degeneration as well. Arteriole narrowing results from extended increased intraluminal pressure and is associated with hypertensive retinal changes. Papilledema is an ominous finding resulting from increased intracranial pressure.

A 41-year-old female has been taking large doses of ibuprofen for the past 2 months owing to osteoarthritis of the hands. She presents today complaining of gnawing upper abdominal pain. What is the underlying mechanism behind the most likely diagnosis? a.Enhanced mucosal blood flow b.Diminished prostaglandin production c.Increased lower esophageal sphincter tone d.Decreased lower esophageal sphincter tone e.Irregular and reversed peristaltic motion

NSAIDs, such as ibuprofen, reduce the production of prostaglandins, which are protective against gastric inflammation. Enhanced mucosal blood flow is a mucosal defense mechanism. Decreased lower esophageal sphincter tone leads to gastroesophageal reflux disease. Increased lower esophageal tone typically occurs in achalasia and results in difficulty swallowing or regurgitation. NSAIDs do not affect peristaltic motion.

A 25-year-old female presents with irregular menses and weight gain. What other physical finding would increase your suspicion for PCOS? a.Dry skin b.Brittle nails c.Hirsutism d.Hypotension

PCOS accounts for about 30% of cases of amenorrhea. The diagnosis is made based on two of these three criteria: oligo- or anovulation, clinical and/or biochemical signs of hyperandrogenism, polycystic ovaries, and exclusion of other etiologies. Approximately 50% of females with PCOS are obese and report irregular menstrual cycles, acne, and hirsutism. Oily skin, not dry, is seen in PCOS, elevated BP, and no hypotension could be seen as features of metabolic syndrome. Nails are typically not affected in patients with PCOS.

A 62-year-old female presents with anterior shoulder pain. She states it is worse when she tries to put dishes in the cupboard above her head. She denies any notable weakness. Which of the following will most likely be positive on physical examination? a.Pain when the shoulder and elbow are at 90 degrees and the shoulder is internally rotated b.Pain when touching the hand to the opposite shoulder c.Pain with pressure over the acromioclavicular joint d.Pain with resisted supination of the forearm e.Pain or apprehension with external rotation of the injured shoulder when it is abducted to 90 degrees and elbow is flexed to 90 degrees

Pain when the shoulder and elbow are at 90 degrees and the shoulder is internally rotated is the Hawkins-Kennedy impingement sign. Rotator cuff impingement is the most likely diagnosis based on the presentation. Pain when touching the hand to the opposite shoulder is the crossover test and would be positive in a patient with acromioclavicular joint arthritis. Pain with pressure over the acromioclavicular joint is also indicative of acromioclavicular joint arthritis. Pain with resisted supination of the forearm is Yergason test, which is usually positive in patients with biceps tendinosis. Pain or apprehension with external rotation of the shoulder when it is abducted, and the elbow is flexed is positive in a patient with recurrent shoulder dislocations.

A 20-year-old female with no known underlying cardiopulmonary disease presents after four recent episodes of near syncope. During these episodes, she was mildly short of breath and was aware of her heart beating rapidly but not erratically. When she sat down and put her head between her knees, she felt better, and her heart slowed down. She has no personal or family history of cardiac problems. ECG done at this visit is entirely normal. What is the most likely diagnosis? a.Atrial fibrillation b.Atrial flutter c.Paroxysmal supraventricular tachycardia d.Ventricular premature beats e.Ventricular fibrillation

Paroxysmal supraventricular tachycardia (PSVT) is common and occurs in patients without structural heart disease. Paroxysms may be interrupted by maneuvers that increase vagal tone. Atrial fibrillation is more common in older individuals and is associated with an irregular rhythm. Atrial flutter is most often seen in persons with chronic obstructive pulmonary disease (COPD) or heart disease. Ventricular premature beats are usually experienced as a sensation of the heart skipping beats. Ventricular fibrillation is a fatal rhythm and therefore not a choice.

An 82-year-old bedbound male with advanced dementia is brought to the office by his caretaker. The caretaker reports finding a "wound" on his left buttock. On physical examination, a 5-cm × 5-cm ulcer is seen that extends through the epidermis and penetrates into, but not through, the dermis. Which stage of pressure injury does this describe? a.Stage 1 b.Stage 2 c.Stage 3 d.Stage 4 e.Unstageable

Patients who are immobile are at greatest risk for pressure injury. Stage 1 pressure injury is characterized by nonblanchable erythema of intact skin. Stage 2 pressure injury is characterized by partial-thickness skin loss with exposed dermis. Stage 3 pressure injury is characterized by full-thickness skin loss, and stage 4 pressure injuries are characterized by full-thickness skin and tissue loss. Unstageable pressure injuries are characterized by obscured full-thickness skin and tissue loss.

A 6-year-old presents with intermittent episodes of wheezing. Examination reveals expiratory wheezing, no rales or rhonchi. She is developing well and is cooperative with the examination. She responds well to an albuterol treatment. What is the most effective method to monitor symptoms at home? a.Peak expiratory flow rate b.Respiratory rate c.Pulse oximetry d.Blood pressure e.Spirometry

Peak expiratory flow rate can be monitored at home using an inexpensive handheld device. Respiratory rate is unreliable and too subjective for monitoring asthma. Pulse oximetry may be normal with mild asthma. The only blood pressure abnormality related to asthma is pulsus paradoxus, which may occur with severe asthma; however, its absence does not correlate with asthma control. Spirometry is helpful in the diagnosis of asthma and COPD but is not used to monitor symptoms at home.

A 17-year-old male presents with a wide-scale, symmetrical papular eruption over his trunk. Lesions developed over several days and align along the skin folds. He had a mild upper respiratory infection last week, which resolved in 2 days. What is the most likely diagnosis? a.Actinic keratosis b.Guttate psoriasis c.Impetigo d.Systemic lupus erythematosus e.Pityriasis rosea

Pityriasis rosea, the most likely diagnosis, is characterized by an initial solitary herald patch that appears several days to weeks before the generalized rash. Sometimes, an upper respiratory prodrome is present before the rash. The rash consists of oval scaly lesions that develop along cleavage lines (Langer lines), said to resemble a "Christmas tree" pattern, and may be mildly pruritic. Because the disorder is more common in the spring and fall and concurrent household cases have been recorded, it is thought to have a viral etiology. The disorder is self-limited and only requires supportive care. Actinic keratosis presents as a flesh-colored macule or papule with a sandpaper feel. They occur in sun-exposed areas. Guttate psoriasis is characterized by teardrop-shaped, scaly lesions that most often develop after a streptococcal infection. Honey-colored, crusted lesions localized to an area of infection characterize impetigo. Systemic lupus erythematosus may include a butterfly rash on the face. It is far more common in females than males.

A 36-year-old female with a history of rheumatoid arthritis was found to have anemia on her most recent blood work. Her hemoglobin was 9.8 (12 to 16 g per dL), hematocrit 32.4 (37% to 47%), and MCV 86 (80 to 95 fL). Which of the following describes the etiology for the most likely type of anemia this patient has? a.Chronic, persistent blood loss b.Erythropoietin cell receptor resistance c.Iron transportation blockage because of increased hepcidin d.Increased destruction of red blood cells e.Loss of iron stores because of chronic inflammation

Rheumatoid arthritis is a systemic disease that causes anemia of chronic disease. Anemia of chronic disease is further characterized by anemia of inflammation, anemia of organ failure, or anemia of older adults. In this case, rheumatoid arthritis is associated with anemia of inflammation. Anemia of chronic disease is most commonly caused by blockage of iron transportation owing to increased hepcidin. The hepcidin blocks iron absorption from the gut and blocks the release of stored iron from the bone marrow. Defects in the erythropoietin receptor may produce erythroleukemia and familial erythrocytosis. Overproduction of red blood cells increases the chance of an adverse cardiovascular event, such as thrombosis and stroke. Increased destruction of red blood cells is hemolytic anemia. Chronic blood loss leads to anemia because of low volume or dilution and will result in microcytic hypochromic anemia.

A 2-year-old male is brought to the pediatrician by his parents with recurrent episodes of hemarthrosis. On physical examination, the right knee joint is erythematous and swollen. The parents deny any trauma or injury. Labs reveal a low factor VIII level and normal platelets. Which of the following is the most likely diagnosis? a.Mild hemophilia A b.Mild hemophilia B c.Severe hemophilia A d.Severe hemophilia B

Severe hemophilia A is a congenital deficiency of factor VIII, and severe hemophilia B is a congenital deficiency of factor IX. Severe hemophilia corresponds to factor VIII activity <1% in hemophilia A and factor IX activity <1% in hemophilia B. Hemarthroses are the most common finding in severe hemophilia. Patients with mild hemophilia usually do not experience abnormal bleeding unless they have surgery or sustain trauma.

A 58-year-old postmenopausal female with mild obesity describes a 2-year history of increasing sensation of vaginal fullness, urinary stress incontinence, inability to fully empty urinary bladder, and dyspareunia. Past obstetric history includes four vaginal deliveries. She has no history of pelvic surgery or connective tissue disorders. Pelvic examination reveals poor pelvic floor muscle tone and an anterior soft fullness descending down level to the plane of the hymen. What is the staging of this finding via the Baden-Walker system? a.Stage 0 b.Stage I c.Stage II d.Stage III

Stage II prolapse of pelvic organs, this patient, is defined as ≤ 1 cm proximal or distal to the plane of the hymen. Stage I prolapse of pelvic organs is defined as the presenting part >1 cm above the plane of the hymen. Stage III prolapse is defined as >1 cm below the plane of the hymen. Stage IV is defined as maximal descent.

A 12-year-old male woke suddenly this morning with severe scrotal pain and edema of the scrotal sac that has continued for 2 hours. He has had similar pain before, but it subsided without intervention. Physical examination reveals a tender, swollen, retracted testis. A routine urine examination is normal. Which of the following is the most appropriate next step? a.Diuretics and analgesia b.Scrotal support and ice packs c.Immediate surgical intervention d.Initiation of antibiotic therapy e.Tc-99m pertechnetate scan

Testicular torsion is most common in adolescent boys. It is a surgical emergency; delaying surgical intervention beyond 4 to 6 hours can lead to testicular infarction and subsequent infertility. Tc-99m pertechnetate scans will confirm the presence of torsion; however, Doppler studies are quicker, less invasive, and just as effective to confirm the diagnosis. Ice packs cause further vasoconstriction, thereby increasing the risk of infarction. Antibiotic therapy has no place in the treatment of torsion. Diuretics are not indicated.

A 72-year-old widowed woman presents to the office with the odor of alcohol on her breath on repeated office visits. When asked about alcohol use, she flatly denies it. Which of the following laboratory studies would best support the diagnosis of alcohol abuse in this patient?

The GGTP is generally elevated in alcohol abuse and is one of the best early indicators of alcohol abuse. With alcohol abuse, the complete blood count (CBC) more often reveals macrocytosis. The enlarged mean corpuscular volume (MCV) results from the toxic effect of alcohol on erythrocyte development and nutritional deficiencies. Liver transaminases (ALT and AST) are often elevated but may be normal or low in advanced liver disease and are not supportive in this case. Long-term alcohol use and cirrhosis are often accompanied by thrombocytopenia and coagulation disorders. Although HDLs tend to be elevated in alcohol abuse, this finding is not specific to alcohol abuse and may indicate healthy lipid status.

A neonate is found to have right ventricular hypertrophy, right ventricular outflow obstruction, and an overriding aorta via cardiac imaging. What additional finding would you expect to see in this child? a.Left atrial septal defect b.Right atrial septal defect c.Discrepancy in upper and lower extremity blood pressure d.Left bundle branch block e.Ventricular septal defect

The finding of ventricular septal defect completes the diagnostic criteria for tetralogy of Fallot, which is the diagnosis here. Although ECG findings are not of great significance, usually right bundle branch block occurs in tetralogy of Fallot. Dissimilar blood pressure readings of the extremities are pathognomonic of coarctation of the aorta. The defect in tetralogy of Fallot is in the ventricle, not the atria, eliminating those choices.

A 32-year-old female was doing yard work and sustained a dirty cut. The soil is known to be contaminated with clostridial spores. If her immunization status is subtherapeutic, what is the likely initial manifestation of disease? a.Altered consciousness b.Diplopia and lack of accommodation c.Jaw and neck stiffness d.Pain and tingling at the site e.Respiratory distress

The initial manifestation of tetanus is pain and tingling at the site of the contaminated wound. Jaw and neck stiffness and respiratory distress are late manifestations of tetanus. The patient with tetanus remains fully conscious and alert with no sensory deficit. Diplopia and lack of accommodation occur in botulism, a foodborne clostridial disease.

A 25-year-old male presents to the clinic complaining of sudden onset of dysuria, urinary frequency, shaking chills, and low back pain. Examination reveals fever, normal genitalia, and tender perineum. Which of the following physical examination techniques is contraindicated at this time? a.Expression of penile discharge b.Palpation of the abdomen with Valsalva c.Percussion of the urinary bladder d.Rectal examination of the prostate e.Assessment for costovertebral angle tenderness

The major risk of performing a rectal examination in a patient with acute prostatitis is sepsis. Manipulation of the infected gland may cause dissemination of the pathogen into the bloodstream. Prostatitis does not cause urethral discharge. Palpation of the abdomen, percussion of the bladder, and assessment for CVA tenderness are not contraindicated in prostatitis.

A 42-year-old male presents for treatment of male pattern baldness. Finasteride is prescribed. What side effect should he be warned about? a.Change in hair color b.Impaired vision, including color blindness c.Loss of libido d.Pernicious anemia e.Sleep disturbance

The most commonly reported side effect of treatment with finasteride is loss of libido along with possible impotence. Change in hair color, impaired vision, pernicious anemia, and sleep disturbance are not associated with finasteride treatment.

A 79-year-old female with a history of type 2 diabetes, hypertension, and tobacco abuse presents complaining of having three episodes of severe epigastric and periumbilical pain over the past month. All three episodes occurred after eating and lasted for 1 hour before resolving on their own. On physical examination, the abdomen is soft and nontender to palpation. Which of the following is the best test to confirm the suspected diagnosis? a.Abdominal ultrasound b.CTA of the abdomen c.Esophagogastroduodenoscopy (EGD) d.Upper GI series e.Plain film of the abdomen

The most likely diagnosis is intestinal ischemia, which is the result of atherosclerosis and hypertension, both brought on through the combination of diet, smoking, and diabetes—atherosclerotic risk factors. CTA or MRA is the best test to determine the presence of intestinal ischemia. These tests can show narrowing of the proximal visceral vessels. An abdominal ultrasound is a noninvasive procedure used to assess the organs and structures within the abdomen, including the liver, gallbladder, pancreas, bile ducts, spleen, and abdominal aorta. An EGD is used to examine the lining of the esophagus, stomach, and duodenum; it is used to detect GERD, gastritis, and peptic ulcer disease. An upper GI series is a group of x-rays used to look at the esophagus, stomach, and duodenum and tell how well they are working. A plain film of the abdomen is an x-ray used to assess bowel obstruction or intestinal perforation.

A 43-year-old female complains of several brief episodes of lancinating facial pain that starts at the left side of her mouth and radiates to the eye and ear occurring the past 3 days. Further questioning reveals that which of the following would likely trigger the episodes? a.Chewing b.Menses c.Fever d.Herpes outbreak e.Sleep deprivation

The pain of trigeminal neuralgia (tic douloureux) is triggered by touch, movement such as talking or chewing, or cold drafts. Fever is not associated with trigeminal neuralgia. Neither sleep deprivation nor menses precipitate trigeminal neuralgia. Herpes outbreaks are the most common cause of erythema multiforme minor.

A 5-year-old male presents with a lacy red rash on his face and a pink macular rash on his chest. His parent denies fever, cough, or anorexia. What is the most likely pathogen causing his symptoms?

The patient has erythema infectiosum, also known as fifth disease or "slapped cheek." It is caused by human parvovirus B19. Human herpesvirus 6 (or 7) causes roseola, which also has a pink macular rash, but is accompanied by a prodromal period. Measles has a prodromal period of fever, cough, and coryza. Measles is also associated with clustered, white lesions on the buccal mucosa known as Koplik spots, followed by a maculopapular exanthem that starts on face and goes to the extremities. By contrast, rubella has a maculopapular rash that starts on the head and progresses toward the feet; it has no prodromal period. Mumps does not manifest a rash.

A 61-year-old male presents complaining of frequent nosebleeds, headaches, dizziness, and pruritis after showering for the past six months. On physical examination, his face has a ruddy appearance, and the spleen is palpable. Laboratory evaluation reveals hematocrit of 56% (normal 42% to 52%), WBC 12,000 per mm3 (normal 5,000 to 10,000/mm3) and increased platelets. What is the recommended treatment for the most likely diagnosis? a.Monthly B12 injections b.Serial phlebotomy c.Chelation d.Oral hydroxyurea e.Avoidance of oxidative drugs

The patient has polycythemia vera, the treatment for which is serial phlebotomy. One unit of blood is removed weekly until the hematocrit is <45%. Monthly B12 injections is the treatment for macrocytic anemia caused by B12 deficiency; and chelation is the treatment for lead toxicity. Oral hydroxyurea is the treatment for sickle cell disease which can cause pain crises. Avoidance of oxidative drugs is indicated in G6PD deficiency, which presents with episodic hemolysis.

A patient presents to the emergency department with headache, lethargy, and dizziness. He is found to have a serum sodium of 110 mEq per L. His BP is 124/82 mm Hg. He has no peripheral edema. Serum osmolality is 250 mOsm per kg. Urine sodium is 50 mEq per L. Which of the following is the most likely cause of the patient's condition? a.Hyperglycemia b.Syndrome of inappropriate antidiuretic hormone (SIADH) c.Congestive heart failure d.Acute renal failure e.Hyperlipidemia

The patient is hypotonic (serum osmolality <280 mOsm/kg) and hyponatremia (serum sodium <135 mEq/L) with euvolemia (normal BP and no edema). The causes of hypotonic hyponatremia with euvolemia include hypothyroidism, glucocorticoid excess, and SIADH. Hyperglycemia causes a high plasma osmolality. Congestive heart failure will show signs of extracellular fluid volume overload (i.e., edema). Acute renal failure will also have signs of hypervolemia. Hyperlipidemia may be associated with isotonic hyponatremia.

A 30-year-old female with a history of PCOS presents to the clinic with painful lesions on her face, neck, and upper trunk for the past 2 years. She reports being on oral antibiotics in the past without significant improvement. On physical examination, the skin has a rough texture with a mixture of tiny, flesh-colored, white and black noninflamed superficial papules and tender papulopustular papules. She will be started on the appropriate medication for her condition. What lab values needed to be assessed before administering treatment? a.AST b.C-peptide c.Hgb d.Sodium e.HbgA1c

The patient is presenting with severe acne vulgaris. The treatment of choice for severe acne is isotretinoin. Laboratory tests including total cholesterol levels, triglyceride levels, and liver enzyme tests (AST/ALT) are recommended in patients before treatment and after achieving therapeutic dosing. Hgb is used to assess anemia. C-peptide measures endogenous insulin production. HgbA1c measures the amount of blood glucose attached to hemoglobin.

A healthy 66-year-old presents as a new patient. She has no complaints, no significant medical history, and does not smoke or drink alcohol. She states she gets the flu shot annually and that she has already received the vaccine this year. What is your recommendation regarding immunizations? a.After 10 consecutive seasons of vaccine, annual flu vaccine is no longer needed b.Annual flu shots should be encouraged until age 75 years c.Administer the pneumococcal polysaccharide vaccine (PPSV 23) d.Obtain pneumococcal titers to assess for protection e.Administer the pneumococcal conjugate vaccine (PCV 13)

The patient should be given the PCV 13 vaccine at this time. Per the CDC 2017 vaccination guidelines, PCV 13 should be given to all adults aged 65 and older, with the PPSV 23 vaccine given 1 year later. Titers are not routinely drawn to assess for immunity. Adults with certain chronic diseases follow a different schedule of revaccination. Flu shot should be administered annually throughout the life span.

An 18-year-old otherwise healthy female presents with cough, mild dyspnea, and fever for 3 days. Her examination is normal except for mild rhonchi heard on expiration. Her chest x-ray is normal. What is the treatment of choice for the most likely diagnosis? a.Azithromycin b.Amoxicillin c.Levofloxacin d.Vancomycin e.Supportive measures

The patient's most likely diagnosis is bronchitis. Over 90% of cases are caused by a virus, and supportive measures in young otherwise healthy patients are appropriate. Azithromycin is used to treat CAP. Fluoroquinolones are first-line treatment for CAP in patients with underlying heart, lung, liver, or renal disease or diabetes mellitus, alcoholism malignancy, asplenia, or immune suppression. Amoxicillin is appropriate for treating streptococcal pharyngitis. Vancomycin should be reserved for patients who are very ill or if methicillin-resistant Staphylococcus aureus is suspected.

A 6-year-old child presents with cold symptoms and "pinkeye" for 2 days. Examination reveals injected conjunctiva, watery or mucoserous eye discharge, clear rhinorrhea, and tender preauricular adenopathy. What is the next step in management? a.Cold compresses b.Cycloplegics c.Topical antibiotic d.Topical antiviral e.Topical corticosteroid

The presentation is likely viral conjunctivitis; the tender preauricular adenopathy is key to diagnosis. There is no specific antiviral treatment, topical or systemic, that is effective for the most common cause, which is adenovirus. A few days of cool to cold compresses typically suffice. Review of personal hygiene measures (do not touch face or mucous membranes, wash often) is also appropriate, as the condition is highly contagious to others through direct contact and fomite contamination. Cycloplegics are appropriate when eye pain is severe and primarily useful to aid in completing an examination on an eye that has undergone trauma; any additional use is discouraged. Topical antiviral treatment is reserved for severe or virulent strains of viral conjunctivitis, such as herpes. Topical antibiotic treatment is not necessary in conjunctivitis that is likely viral. Corticosteroids should be avoided owing to potential sight-threatening complications. Supportive measures for the additional URI symptoms such as antihistamines and fluids can be recommended on a case-by-case basis.

The CDC recommends screening of all pregnant women for group B streptococcal colonization. What is the proper collection protocol? a.First-trimester urine culture and sensitivity b.Late first-trimester culture of endocervix c.Late second-trimester culture of endocervix d.Early third-trimester vaginal culture e.Mid-third-trimester culture of the distal vagina and anorectum

The proper protocol for universal prenatal screening for group B streptococcal colonization per the CDC is to collect a one-time culture with specimens from the distal vagina and the anorectum at 35 to 37 weeks.

A mother brings in her 4-year-old child to assess a lesion on his left malleolus. The lesion is nontender, flesh colored, raised, and has a velvety appearance. What is the most likely cause?

The vast majority of human warts are caused by human papillomavirus types. A.Coxsackievirus causes herpangina, hand-foot-mouth disease, and other syndromes, but not warts. C. Herpes virus breakouts consist of grouped vesicles. B. CMV causes a mononucleosis-like syndrome as well as opportunistic retinitis, esophagitis, pneumonia, and encephalitis in patients who are immunocompromised, such as HIV. E. Adenovirus is associated with upper respiratory infections (URIs) and conjunctivitis, but rarely with skin conditions.

Which of the following glucose-lowering agents acts like an insulin secretagogue that stimulates insulin release by the pancreatic β cells? a.Acarbose b.Canagliflozin c.Glipizide d.Metformin e.Pioglitazone

There are several pathways utilized in glucose-lowering therapies. Glipizide (Glucotrol) is a sulfonylurea that works by increasing insulin secretion from pancreatic β-cells. Pioglitazone (Actos) is a thiazolidinedione (TZD) that decreases insulin resistance and increases glucose utilization by sensitizing peripheral tissues to insulin. α-Glucosidase inhibitors, such as acarbose (Precose), reduce glucose by delaying carbohydrate absorption. Sodium glucose cotransporter 2 (SGLT2) inhibitors, such as canagliflozin (Invokana), block glucose resorption in the proximal renal tubule. Metformin (Glucophage), a biguanide, lowers glucose by decreasing hepatic glucose production and increasing glucose utilization.

A 25-year-old female presents complaining of acute episodes of feeling fearful accompanied by shaking, shortness of breath, nausea, and dizziness. She states that these episodes have occurred several times in the past 2 months, and now she worries constantly that she will have "an episode." She is concerned that she is "losing it" and has been avoiding big social gatherings for the past month. What treatment plan is important to include in educating this patient? a.Avoiding social gatherings will be beneficial b.Treatment with a tricyclic antidepressant will be helpful c.The overall prognosis is not good, and she should expect hospitalization d.There is an increased chance that she may also develop a severe depression e.She should avoid peppermint

There is a 40% chance that this patient with panic disorder may experience a severe depression, and patients should be counseled on the symptoms of depression. Antidepressants, notably selective serotonin reuptake inhibitors (SSRIs), should be considered as part of the overall treatment plan. Tricyclic antidepressants are effective for panic; however, the side effect profile is such that they are rarely used. Mild panic disorders can be treated with psychotherapy alone. The prognosis of panic disorder with treatment is good. Peppermint or chamomile tea may be relaxing and beneficial.

An 8-month-old male infant has had rhinorrhea, sneezing, cough, and low-grade fever for 2 days. On examination, there is nasal flaring, tachypnea, retractions, and wheezes. Chest radiography reveals air trapping and peribronchial thickening. What is the most likely diagnosis? a.Acute bronchiolitis b.Asthma exacerbation c.Cystic fibrosis d.Viral pneumonia e.Croup

These are classic clinical symptoms and signs of acute bronchiolitis, which is defined as upper respiratory symptoms followed by lower respiratory inflammation in a child younger than 12 to 24 months of age; diagnosis is typically based on clinical findings. A diagnosis of asthma is made after a recurrent pattern of reversible bronchoconstriction. Cystic fibrosis may be associated with poor growth, recurrent pulmonary infections, chronic diarrhea, or a family history of the disease. Pneumonia is usually associated with an infiltrate demonstrated on chest radiography with focal crackles or decreased breath sounds on physical examination. Croup has a classic "barking" cough and may have stridor associated with it; radiography reveals subglottic narrowing ("steeple sign").

An 8-month-old male infant has had rhinorrhea, sneezing, cough, and low-grade fever for 2 days. On examination, there is nasal flaring, tachypnea, retractions, and wheezes. Chest radiography reveals air trapping and peribronchial thickening. What is the recommended treatment for the most likely diagnosis? a.Doxycycline b.Tetracycline c.Humidified oxygen and IV fluids d.Corticosteroids e.Azithromycin

These are classic signs and symptoms of acute bronchiolitis. Diagnosis is made based on clinical findings. Supportive measures such as humidified oxygen and IV hydration are the treatments of choice. This is most often caused by respiratory syncytial virus (RSV), and antibiotics are not indicated. Corticosteroids may be helpful in the treatment of croup. Evidence shows that patients with bronchiolitis treated with corticosteroids do no better than those who are not.

A 14-year-old male presents with sudden and worsening sore throat and fever but denies cough. Examination reveals elevated temperature, tender anterior cervical adenopathy, and patchy pharyngeal exudate. What is the most likely diagnosis? Select one: a.Epiglottitis b.Mononucleosis c.Peritonsillar abscess d.Streptococcal pharyngitis e.Herpangina

These clinical features (fever, tender anterior cervical adenopathy, lack of cough, and patchy pharyngotonsillar exudate) complete the classic Centor criteria and strongly suggest group A β-hemolytic streptococcal infection. The patient can be treated with the appropriate antibiotic without further diagnostic testing. Streptococcal pharyngitis typically appears suddenly and intensifies quickly and is more common in children ages 5 to 15 years than in adults. Peritonsillar abscess presents with severe sore throat, usually unilateral, dysphagia, trismus ("lockjaw"), unilateral swollen tonsil with contralateral deviation of the soft palate and uvula, and a muffled ("hot potato") voice. Mononucleosis is suggested by prominent bilateral adenopathy (characteristically involving the posterior cervical nodes), enlarged tonsils with a shaggy white-gray exudate, and petechiae, which may extend into the nasopharynx, especially in young adults. Herpangina presents with fever and petechiae or papules on the soft palate that ulcerate before healing. Epiglottitis should be suspected when a patient presents with a rapidly developing sore throat or when pain with swallowing is out of proportion to minimal findings on physical examination. Stridor, drooling, and tripoding are also commonly associated findings. Epiglottitis is less common today in children secondary to Haemophilus influenzae type b (Hib) vaccination administration.

A female presents complaining of irregular menses and infertility for 1 year. Pelvic ultrasonography reveals multiple smooth, pearl-white ovarian cysts characterized as "oyster ovaries." Which of the following lifestyle modifications would be best to help prevent further cysts from recurring

This patient likely has PCOS, which is often characterized by infertility, hirsutism, and obesity. Forty percent of patients with PCOS are obese. Weight loss is often effective in decreasing the severity of the disease. Moderate weightlifting and aerobic activity are advantageous for a healthy lifestyle, but will not help with the treatment of PCOS unless there is accompanying weight loss. Smoking cessation will not help with the treatment of PCOS. Alcohol intake is not related to polycystic ovarian syndrome (PCOS).

A 42-year-old female presents with a new mass in the upper outer quadrant of her right breast. She notes associated serous nipple discharge and some mild discomfort but no redness to the overlying skin. Evaluation with mammography is indeterminant with dense breast tissue. Ultrasonography does not support the diagnosis of cystic lesion. The mass persists over the next 2 months. What is the next step? a.Give a trial of antibiotics to relieve associated mastitis b.Follow the mass with serial monthly MRI to assess for changes c.Perform needle aspiration to determine cytology d.Recommend simple mastectomy to prevent the spread of malignancy e.Evaluate with thermal ultrasonography

This case most likely represents fibrocystic breast changes. However, if a mass persists through cycles, further workup is recommended. Fine-needle aspiration (FNA) is used to evaluate the cytology which is needed for definitive diagnosis. If the mass persists and the FNA is negative, the mass should be excised to evaluate for malignancy. MRI and ultrasonography do not provide a histologic evaluation. A simple mastectomy is for the treatment, not the diagnosis, of breast cancer. This patient does not present with symptoms consistent with breast infection (acute onset, redness, and tenderness). Mastitis most commonly occurs in the puerperium.

A 9-year-old male complains of abrupt onset of sore throat and fever without cough or dyspnea. Examination reveals tender cervical adenopathy and pharyngeal exudate. What is the recommended management for the most likely diagnosis? a.Obtain culture; treat based on results b.Saltwater gargles and acetaminophen c.Treat with amoxicillin d.Treat with tetracycline e.Check CBC with differential

This child has four Centor points: fever, absence of cough, tender adenopathy, and pharyngeal exudate. The recommendation is to treat the diagnosis; amoxicillin is the first-line treatment unless otherwise contraindicated. A culture is not necessary when the Centor criteria are greater than three points. Saltwater gargles and acetaminophen will provide symptomatic relief, but will not eradicate the streptococcal organisms. Tetracyclines are contraindicated in children because of staining of teeth and bones. A CBC with differential is nonspecific and is not necessary; the diagnosis is made primarily on clinical grounds with confirmatory diagnostic studies when indicated.

A 15-year-old female complains that she has never had menses. She is short in stature and has a webbed neck and wide-spaced nipples. Initial lab work shows a high FSH. Which of the following best confirms the diagnosis? a.Estrogen levels b.Karyotype c.LH levels d.Ultrasonography e.Testosterone levels

This is a classic picture of Turner syndrome (gonadal dysgenesis), which is a frequent cause of primary amenorrhea. Karyotyping will show a 45XO pattern. Estrogen levels will be abnormally low, but are not diagnostic. LH levels will be elevated, but are not helpful in diagnosing Turner syndrome. Ultrasonography will show a gonadal streak, which represents misshapen, functionless ovarian tissue. Although present in Turner syndrome, gonadal streaks are also seen with other genetic mutations and, therefore, are not diagnostic. Testosterone levels are not influenced by Turner syndrome.

A 45-year-old male who is recuperating from influenza suddenly develops sharp substernal pain in his chest. He cannot get comfortable lying down but is able to get some relief by sitting forward. He also feels as if he cannot get a deep breath, but has no other symptoms other than residual cough from the flu. On cardiac auscultation, a squeaking sound is heard. His ECG demonstrates ST and T wave changes in all leads. CXR is unremarkable. What is the most likely diagnosis? a.Acute coronary event b.Angina pectoris c.Costochondritis d.Inflammatory pericarditis e.Dissecting thoracic aneurysm

This is a classic presentation of acute pericarditis following a viral infection, which is the most common cause of inflammatory pericarditis. The pain of an acute coronary event is more likely to be crushing or squeezing than sharp and is not relieved by sitting. Anginal pain is typically described as squeezing, tightness, or heaviness; it is characteristically worsened with exertion. Dissecting thoracic aneurysm pain is acute, tearing is not changed with position. Costochondritis may present with sharp pain associated with movement, but has no associated ECG changes.

A 76-year-old female presents with a 2-day history of headache and jaw pain, which is exacerbated by chewing. Erythrocyte sedimentation rate (ESR) is 60 mm per hour. What is the immediate treatment? a.Cyclophosphamide b.Fluid resuscitation c.Oral prednisone d.Pulsed methylprednisolone e.Methotrexate

This is a patient presenting with giant cell arteritis. Immediate treatment with prednisone starting at 60 mg per day is therapeutic and may prevent permanent blindness associated with giant cell arteritis. Fluids are part of the overall patient management but do not directly help relieve the pathology of giant cell arteritis. Pulsed steroids are indicated for critically ill patients with polyarteritis nodosa or patients with giant cell arteritis who have developed visual loss. Cyclophosphamide and methotrexate are anticancer chemotherapeutic agents and are not indicated in giant cell arteritis.

A 52-year-old female presents complaining of constant lower extremity pain. Physical examination reveals dilated, tortuous, elongated veins of her right thigh and leg, most of which are <4 mm in diameter. Which of the following interventions is indicated for definitive treatment for this patient? a.Compression sclerotherapy b.Greater saphenous vein stripping c.Leg elevation with bed rest d.Sequential compression devices e.Warfarin

This is a patient presenting with symptomatic varicose veins. As a result, compression sclerotherapy produces permanent fibrosis in small veins; it typically relieves all symptoms and is, therefore, the definitive treatment. Stripping is reserved for large vessels as a second resort after ablation therapy. Leg elevation and rest are palliative, not definitive. Sequential compression devices are used for nonambulatory patients, usually in the acute care setting, as prophylaxis against thrombosis. Warfarin is used in the treatment of DVT, not superficial varicose veins of the lower extremity.

A 72-year-old recently widowed female is hospitalized with an acute anterior wall myocardial infarction. Echocardiography reveals left ventricular apical ballooning, yet cardiac catheterization reveals clear coronary arteries. What is the most likely diagnosis? a.Aortic aneurysm b.Cardiac tamponade c.Prinzmetal angina d.Takotsubo cardiomyopathy e.Acute pericarditis

This is the classic description of Takotsubo cardiomyopathy (literally "octopus bottle"), found in postmenopausal women often occurring after a major discharge of catecholamines. Although coronary arteries are also usually clear in Prinzmetal angina, the ventricular enlargement, if present, would be more uniform. Imaging would show widening of the aorta in aortic aneurysm. In tamponade, the echocardiography would demonstrate pericardial fluid and additional cardiac pressure changes. With pericarditis, ST elevation would not be limited to anterior wall leads only; rather, ST elevation would be present in both limb and precordial leads.

A 12-year-old girl is found to have elevated BP readings on three separate occasions. Her BP is normal in her legs, but femoral pulsations are weak. ECG demonstrates left ventricular hypertrophy. What is the most likely cause of her hypertension? a.Atrial septal defect b.Coarctation of the aorta c.Pheochromocytoma d.ToF e.Ventricular septal defect

This is the classic description of coarctation of the aorta as a cause of secondary hypertension. Large ventricular septal defects may cause arrhythmias, but are not associated with hypertension. Atrial septal defect, if undiscovered and/or untreated in childhood, may cause dyspnea and fatigue in older children, which may lead to sustained atrial arrhythmias, stroke, or heart failure. ToF includes overriding aorta, pulmonic stenosis, right ventricular hypertrophy, and ventricular septal defect; it is not associated with secondary hypertension. Untreated, older children with ToF are at risk for pulmonary valve problems and congestive heart failure. After surgical correction, older children are at risk for endocarditis, arrhythmias, and coronary artery disease. Pheochromocytoma is a cause of secondary hypertension but presents with flushing, headaches, and fluctuating BPs.

A 75-year-old male with a history of hypertension, hyperlipidemia, and tobacco abuse presents complaining of dyspnea on exertion, which he has contributed to aging, and a recent episode of syncope when walking his dog. Which of the following best describes the murmur that would likely be found on physical examination in support of the most likely diagnosis? a.Accentuated S1 with an opening snap following S2, heard best at the left sternal border and apex b.A diastolic rumbling murmur, heard best at the left lower sternal margin and the xiphoid, augmented during inspiration c.A low-pitched, blowing decrescendo diastolic murmur, heard best at the third left intercostal space along the left sternal border d.A systolic ejection murmur that peaks later in systole, heard best at the second intercostal area with radiation to the neck and apex e.Loud, high-frequency, systolic murmur accompanied by a palpable thrill

This is the typical physical examination finding in aortic stenosis, which is the most common acquired valve disorder in the elderly and is associated with hypertension, elevated lipids, a bicuspid aortic valve, and smoking. An accentuated S1 with an opening snap following S2 heard best at the apex and, possibly, the left sternal border is indicative of mitral stenosis. A diastolic rumbling murmur following an audible opening snap is heard best at the left lower sternal margin is indicative of tricuspid stenosis and typically presents with right heart failure; this murmur of tricuspid stenosis will increase with inspiration. A low-pitched, blowing decrescendo diastolic murmur heard in the left second to fourth intercostal spaces is the typical finding with aortic regurgitation, which is less commonly seen today because of the decreased prevalence of rheumatic heart disease. A loud, high-frequency blowing, holosystolic murmur with radiation to the left axilla indicates mitral regurgitation.

Physical examination of a 60-year-old male reveals mild perioral cyanosis with a normal respiratory rate and no use of accessory muscles. Chest percussion reveals resonance; auscultation demonstrates wheezes and coarse rhonchi that change in location and intensity after a cough. What is the most likely diagnosis? a.Chronic asthma b.Chronic bronchitis c.Community-acquired pneumonia d.Emphysema e.Viral upper respiratory infection

This is the typical picture of chronic bronchitis-predominant COPD. Asthmatics are not cyanotic unless the disease is very severe; they typically have an increased respiratory rate and demonstrate expiratory wheezes rather than rhonchi. Patients with emphysema-predominant COPD display tachypnea, use of accessory muscles, and diminished breath sounds but typically do not demonstrate rhonchi; cyanosis may develop late in the disease. Patients with pneumonia typically present with productive cough and increased respiratory rate; chest percussion may be dull because of an infiltrate. A viral URI may have some of the features described but is milder and does not usually cause cyanosis.

A 10-year-old male with obesity presents with knee pain for the past few weeks. He states the pain is worse when he is active. Knee examination is normal, but there is limited internal rotation of his hip. X-ray shows posteromedial angulation of the femoral epiphysis. Klein line does not pass through any portion of the femoral head. What is the definitive treatment for this patient? a.Abduction splint b.Long leg cast c.Non-weight bearing for 6 weeks d.Surgical pinning e.Follow-up in 6 months

This patient has SCFE that must be addressed as soon as possible with surgery. The initial treatment for a patient with SCFE is non-weight bearing until they can be seen by an orthopedic surgeon for surgical pinning. Surgical pinning is the definitive treatment in a patient with SCFE. The epiphysis is fractured and must be held in place while the fracture heals. It has not been found beneficial to reduce the fracture and put the femoral head back into anatomic alignment. An abduction splint is used to treat patients with Legg-Calvé-Perthes disease or avascular necrosis of the femoral head. A long leg cast does not immobilize the hip joint, which is what is needed in a patient with SCFE.

A patient is admitted to the ICU after developing sepsis. Approximately 12 hours after the onset of sepsis, he develops severe dyspnea. Physical examination shows severe hypoxia, tachypnea, frothy pink sputum, and diffuse crackles. What is the treatment of choice for this patient? a.Levofloxacin b.Intubation and low-level positive end-expiratory pressure (PEEP) c.Intubation and high-level PEEP d.Bilevel positive airway pressure (BiPAP) e.High-flow oxygen via nonrebreather

This patient has developed ARDS. Sepsis is the most common cause of ARDS. It is the result of increased permeability of the alveolar-capillary membrane, resulting in protein-rich pulmonary edema. Mortality rate is high. Treatment is intubation with low-level PEEP. High-level PEEP can cause the condition to worsen. Levofloxacin may be appropriate to treat the infection, but will not address ARDS. BiPAP may be used in acute exacerbations of asthma or COPD. High-flow oxygen via nonrebreather is not sufficient to address ARDS.

A college student who has been under high stress studying for examinations and applying to graduate school presents with facial pain and stiffness. The symptoms first began about 1 week ago. He states his eyes and mouth have become so dry that it is affecting his ability to study or eat. Examination reveals inability to close the left eye and an asymmetric grimace. What is the recommended treatment for the most likely diagnosis? a.IVIG b.Diphenhydramine c.Low-salt diet d.Lubricating eye drops e.Prednisone

This patient has developed Bell palsy. Lubricating drops are needed to prevent corneal abrasions or keratitis. Prednisone is beneficial but only if started within the first 3 to 5 days of symptoms. IVIG is not beneficial in this typically self-limiting disease. Low-salt diet is beneficial in Ménière disease. Diphenhydramine is not helpful and may worsen the dry eye feeling.

A 46-year-old female presents to the emergency department with a fiery red, painful rash with well-demarcated margins on her left cheek. The affected area is shiny with an orange peel-like surface. She reports feeling feverish and experiencing chills over the past 24 hours. What is the most likely diagnosis? a.Bowen disease b.Cellulitis c.Erysipelas d.Kaposi sarcoma e.Toxic epidermal necrolysis

This patient has erysipelas, a fast-spreading and potentially serious skin infection that involves the dermis and lymphatics, most often caused by streptococcal bacteria, typically β-hemolytic streptococcal infections. It is a more superficial infection than cellulitis, which affects the dermis as well but can extend into the subcutaneous tissues. Cellulitis is generally not as fast spreading as erysipelas, which can progress very quickly if not treated in a timely manner. Patients with erysipelas can experience prodromal systemic symptoms, such as chills, fatigue, anorexia, vomiting, and feeling feverish up to 48 hours before the onset of rash. Bowen disease is squamous cell carcinoma in situ. Kaposi sarcoma is a purplish-to-brown, firm cancerous lesion associated with HIV/AIDS infections. Toxic epidermal necrolysis is a serious and life-threatening systemic desquamation disorder, by definition, covering >30% of total body surface area and is typically triggered by a severe drug reaction.

A 46-year-old female presents to the emergency department with a fiery red, painful rash with well-demarcated margins on her left cheek. The patient is examined and released on oral penicillin treatment. What should be included in patient education before discharge? a.Avoid alcohol during treatment b.Migratory arthralgia may occur c.Visual disturbance is possible d.Rash will desquamate in 5 to 10 days e.Sun exposure will cause worsening

This patient has erysipelas, a fast-spreading and potentially serious skin infection that involves the dermis and lymphatics, most often caused by streptococcal bacteria. The rash of erysipelas typically desquamates in 5 to 10 days. Arthralgia and visual disturbances are not typical in streptococcal infections. Erysipelas is not sun sensitive nor is penicillin a photosensitizing medication. Alcohol intake is a concern when taking metronidazole, but not penicillin.

A 27-year-old schoolteacher reports 48 hours after placement of screening PPD. She has no known risk factors. An 11-mm area of induration is palpated. Previous annual screenings have all been negative. CXR is negative. What is the next step in diagnosis? a.No treatment necessary b.Treat with INH for 9 months c.Treat with INH, RIF, and PZA until cultures return d.Treat with RIF for 4 months e.Induce sputum for testing

This patient is a recent converter. She was likely exposed and harbors tuberculosis organisms. Treatment will prevent active disease. Inducing sputum for diagnosis is not necessary in latent tuberculosis. Optimal treatment for latent tuberculosis in people at risk of exposure and who stand to benefit from treatment is INH for 9 months; alternative regimen is RIF and PZA for 2 to 3 months. Treatment with RIF alone for 4 months is acceptable for those who cannot tolerate INH. Triple-drug therapy is recommended for active tuberculosis.

A 60-year-old male is found to have a new systolic ejection murmur during his annual physical examination. The murmur is heard best in the right second interspace and radiates to his neck. Other aspects of the physical examination are unremarkable. Echocardiography reveals a calcified, bicuspid aortic valve. Which of the following is indicated at this time? a.Bovine valve replacement b.Cardiac catheterization c.Cardiac CT scan d.Temporary valvuloplasty e.Nuclear stress test

This patient is exhibiting signs of aortic stenosis and requires additional studies, including a cardiac catheterization before any plans for future valve replacement surgery. CT scan and nuclear stress test would not provide any useful information in planning for a possible valve replacement. Valve replacement surgery is performed urgently only for symptomatic patients (heart failure, syncope, and angina). Balloon valvuloplasty is less effective and associated with high restenosis rates.

A patient who is 1-month status post-liver transplant develops fever, tachypnea, dyspnea, and a nonproductive cough. Chest x-ray reveals diffuse infiltrates. What is the most likely causative organism of his symptoms? a.Mycoplasma pneumoniae b.Legionella spp. c.Pneumocystis jirovecii d.Bordetella pertussis e.Klebsiella pneumoniae

This patient is immunosuppressed because of recent liver transplantation and is at a higher risk of contracting pneumonia caused by Pneumocystis jirovecii. Mycoplasma pneumoniae is a cause of atypical CAP. Legionella and Klebsiella are more common in alcoholics. Bordetella pertussis causes whooping cough and is associated with coughing fits with a "whoop."

A 49-year-old man who is intermittently homeless and has smoked 1/2 pack per day for 30 years presents with 2 weeks of cough. He states that the cough is constant day and night; at first, it was dry but has become productive of yellow sputum. Yesterday, he noticed blood streaks in the sputum. He wakes up at night feverish and diaphoretic. He appears ill, although vital signs and lung examination are insignificant. What is the most likely diagnosis? a.Asbestosis b.Lung cancer c.Pneumonia d.Tuberculosis e.Emphysema

This patient is presenting with the typical signs and symptoms of acute tuberculosis. Homelessness is a risk factor for transmission. Patients with pneumonia are very unlikely to have a normal lung examination. Asbestosis most commonly occurs in construction workers or automobile mechanics; it presents as insidiously progressive dyspnea. A new cough or change in a chronic cough indicates lung cancer in a smoker; dyspnea, weight loss, and hemoptysis are common, but fever and night sweats are not. Emphysema is not associated with fever and has a more insidious onset; sputum is typically absent or mild and is not bloody (hemoptysis).

A 26-year-old vegan presents complaining of fatigue for 3 months. She also complains of anorexia, diarrhea, and numbness in her hands and feet. On physical examination, her tongue appears swollen, shiny, and erythematous. Which of the following would be expected to be seen on her peripheral blood smear? a.Auer rods b.Bite cells c.Microcytic RBCs d.Hypersegmented neutrophils e.Spherocytes

This patient likely has B12 deficiency, which is common in strict vegans because the main source of vitamin B12 is foods of animal origin. It manifests as megaloblastic anemia where macro-ovalocytes and hypersegmented neutrophils are seen on peripheral smear. Microcytic anemia is more likely iron deficiency that is also seen in vegans but less so because of the abundance of iron foods, such as beans, broccoli, fortified cereals, and grains. Spherocytes are rounded red cells that do not show central pallor; they are caused by defects in the cell membrane. Hereditary spherocytosis is characterized by spherocytes, increased reticulocyte count, and hyperchromic red cells. Bites cells are characteristic of G6PD deficiency. Auer rods are pathognomonic for acute myelogenous leukemia (AML).

A 40-year-old male presents complaining of feeling "down" for the past month. He complains that he is very tired all the time despite sleeping 9 to 10 hours per night. He states he has no energy to do anything and has stopped going out on weekends with friends owing to fatigue and loss of interest. He has also noted a 6-lb weight loss and admits to eating less in the past few weeks. He has had episodes of "sadness" in the past, but they were not as severe. The patient denies thoughts of suicide. What is the best diagnosis? a.Major depressive episode b.Melancholy c.Borderline personality disorder d.Dysthymia e.Agoraphobia

This patient meets the criteria for a major depressive episode. A loss of ability to feel pleasure (anhedonia) is characteristic and leads to fatigue and withdrawal from social interactions. Dysthymia is a pattern of hypomania and subdepressive symptoms. Melancholy characteristically has worse symptoms in the morning, often disrupting sleep. A borderline personality displays disruptive behavior that affects all aspects of life, often resulting in arrest or institutionalization. Agoraphobia is a fear of being in a situation where escape may be difficult; this often leads to avoidance of crowds or of leaving one's home.

A 60-year-old asymptomatic male is found to have a new systolic ejection murmur during his annual physical examination. The murmur is heard best in the right second interspace and radiates to his neck. Echocardiography reveals a calcified, bicuspid aortic valve. Other aspects of the physical examination are normal. Which of the following is indicated at this time? a.Bovine valve replacement b.Cardiac catheterization c.CXR (PA and Lat) d.Cardiac CT scan e.Temporary valvuloplasty

This patient requires additional studies, including a cardiac catheterization before any plans for future valve replacement surgery. Bovine valve surgery is not an option because valve replacement surgery is performed immediately only for symptomatic patients (heart failure, syncope, and angina). Chest x-ray or cardiac CT scan would not provide any additional useful information in planning for a possible valve replacement. Balloon valvuloplasty is less effective than valve replacement and is associated with higher restenosis rates.

A 25-year-old African American female presents with an insidious onset of cough, dyspnea, and chest discomfort. She also reports swelling in her cheeks, dry mouth, and diffuse lymphadenopathy. What lab test result will most likely be found in this patient? a.Leukocytosis b.Neutropenia c.Hypocalcemia d.High angiotensin-converting enzyme e.Low CD4 count

This patient's most likely diagnosis is sarcoidosis. It results in idiopathic inflammation of the affected organs including lung, lymph nodes, and parotid glands. It commonly affects young African American women and Northern European whites. Angiotensin-converting enzyme levels are elevated in 40% to 80% of patients. Sarcoidosis causes leukopenia, not leukocytosis. Eosinophilia is possible, but sarcoidosis is not typically associated with neutropenia. Hypercalcemia may be seen, not hypocalcemia. A low CD4 count is seen in HIV or AIDS, but not in sarcoidosis.

A 53-year-old male has a history of schizophrenia. He has been hospitalized several times over the past 25 years. He has been treated with various antipsychotic medications over the years with mixed results. He has an eighth-grade reading level and requires assistance with financial matters. What is expected on cerebral computed tomography (CT)? a.Ventricular enlargement b.Ring-enhancing central lesion c.Loss of central white matter d.Cortical swelling e.Frontal lobe enlargement

Ventricular enlargement and cortical atrophy are common in chronic schizophrenia, especially in cases with cognitive impairment and poor response to medication. Loss of cortical matter is seen in multiple sclerosis; the pathology is better visualized via MRI. Ring-enhancing central lesions are seen in lymphoma or toxoplasmosis. Cortical swelling or edema can result from multiple etiologies, but not typically characteristic of chronic schizophrenia.

A 6-year-old presents with cold symptoms and "pinkeye" for 2 days. Examination reveals red conjunctiva, watery eye discharge, clear rhinorrhea, and a tender preauricular adenopathy. What is the most likely diagnosis? a.Bacterial conjunctivitis b.Herpes keratitis c.Inclusion conjunctivitis d.Keratoconjunctivitis sicca e.Viral conjunctivitis

Viral conjunctivitis is usually associated with URI symptoms. The conjunctiva is red with copious watery discharge and scanty exudate. Preauricular adenopathy is common, and children are affected more often than adults. Bacterial conjunctivitis typically produces a purulent discharge in which there is usually minimal irritation and no visual impairment, and preauricular adenopathy is rare. Inclusion conjunctivitis is commonly a sexually transmitted disease in adults, typically Chlamydia; a nontender preauricular adenopathy is common. Dry eye syndrome is common, especially in elderly females; it can be caused by a variety of conditions, including lacrimal gland dysfunction, systemic disease (i.e., Sjögren syndrome), and drugs.


संबंधित स्टडी सेट्स

CFA Level 1 Understanding Balance Sheets

View Set

4.7 & 4.8 - Inverse Trigonometric Functions and Applications & Models

View Set

Womens Health/Disorders and Childbearing Health EVOLVE

View Set

Open Economy: Real Interest Rate, Exchange Rate, and Trade Balance

View Set

personal money management chapter 13 (bonds)

View Set

Small Business Management Ch. 4-7

View Set